Noether's theoem and conserved current arbitrariness

  • Thread starter Einj
  • Start date
  • Tags
    Current
In summary, the conserved current is given byJ^{\mu} = \frac{\partial \mathcal{L}}{\partial ( \partial_{\mu} \phi )} \ \Delta \phi - \Lambda^{\mu}
  • #1
Einj
470
59
Hello everyone,
I have a question about conserved currents. Suppose that we have a Lagrangian which is perfectly invariant under a certain infinitesimal transformation of the field ##\phi\to\phi+\alpha\Delta\phi##, i.e. ##\mathcal{L}\to\mathcal{L}##. Now, more generally, the Lagrangian should be invariant up to a total divergence, ##\mathcal{L}\to\mathcal{L}+\alpha\partial_\mu\mathcal{J}^\mu##. This means that in our case ##\partial_\mu\mathcal{J}^\mu=0## and so ##\mathcal{J}^\mu## could be any aribtrary function with vanishing divergence.

Now, the general form for the conserved current is:
$$
j_\mu(x)=\frac{\partial \mathcal{L}}{\partial(\partial^\mu\phi)}\Delta\phi-\mathcal{J}_\mu.
$$

Suppose now that the first term has zero divergence as well because of the equation of motion. Now, it seems to me that in this case, I could simply set the current to be zero by choosing ##\mathcal{J}^\mu=\frac{\partial \mathcal{L}}{\partial(\partial^\mu\phi)}\Delta\phi## since this indeed satisfies ##\partial^\mu\mathcal{J}_\mu=0##.

Am I doing something wrong or is this indeed the case?

Thanks!
 
Physics news on Phys.org
  • #3
Einj said:
Hello everyone,
I have a question about conserved currents. Suppose that we have a Lagrangian which is perfectly invariant under a certain infinitesimal transformation of the field ##\phi\to\phi+\alpha\Delta\phi##, i.e. ##\mathcal{L}\to\mathcal{L}##. Now, more generally, the Lagrangian should be invariant up to a total divergence, ##\mathcal{L}\to\mathcal{L}+\alpha\partial_\mu\mathcal{J}^\mu##. This means that in our case ##\partial_\mu\mathcal{J}^\mu=0## and so ##\mathcal{J}^\mu## could be any aribtrary function with vanishing divergence.

Now, the general form for the conserved current is:
$$
j_\mu(x)=\frac{\partial \mathcal{L}}{\partial(\partial^\mu\phi)}\Delta\phi-\mathcal{J}_\mu.
$$

Suppose now that the first term has zero divergence as well because of the equation of motion. Now, it seems to me that in this case, I could simply set the current to be zero by choosing ##\mathcal{J}^\mu=\frac{\partial \mathcal{L}}{\partial(\partial^\mu\phi)}\Delta\phi## since this indeed satisfies ##\partial^\mu\mathcal{J}_\mu=0##.

Am I doing something wrong or is this indeed the case?

Thanks!

If the transformation [itex]\phi \to \phi + \delta \phi[/itex] leaves some Lagrangian invariant (off-shell): [itex]\mathcal{L} \to \mathcal{L}[/itex] (or [itex]\delta \mathcal{L} = 0[/itex]), then you have (on-shell) conserved current of the form [tex]J^{\mu} = \frac{\partial \mathcal{L}}{\partial ( \partial_{\mu} \phi )} \ \delta \phi ,[/tex] and the transformation is an INTERNAL symmetry transformation. If, under some transformation [itex]\Delta \phi[/itex] the Lagrangian change (off-shell) by a total divergence of some object [itex]\Lambda^{\mu}[/itex]: [itex]\delta \mathcal{L} = \partial_{\mu} \Lambda^{\mu}[/itex], then the (on-shell) conserved current is given by [tex]J^{\mu} = \frac{\partial \mathcal{L}}{\partial ( \partial_{\mu} \phi )} \ \Delta \phi - \Lambda^{\mu} ,[/tex] and the transformation [itex]\Delta \phi[/itex] is SPACE-TIME symmetry transformation. The object [itex]\Lambda^{\mu}[/itex] IS NOT arbitrary. How can it be arbitrary? It is what you get when you transform the Lagrangian. So, [itex]\Lambda^{\mu}[/itex] depends on the Lagrangian, and you can not choose it arbitrarily. For example, under the translation [itex]\Delta \phi = - a^{\mu} \partial_{\mu} \phi[/itex], the Lagrangian transforms like [itex]\delta \mathcal{L} = \partial_{\mu} ( a^{\mu} \mathcal{L} )[/itex]. So, [itex]\Lambda^{\mu} = a^{\mu} \mathcal{L}[/itex]. Is this arbitrary? You are confusing the two type of transformation with the freedom to add a divergence term into the Lagrangian. See, the discussion on pages 7-8 in the PHD below.
Sam
 

Attachments

  • Noether Theorem.pdf
    359.9 KB · Views: 187
  • Like
Likes strangerep and Einj
  • #4
Yes, I understand now. Thanks you very much for your help!
 

1. What is Noether's theorem?

Noether's theorem is a fundamental principle in physics that relates symmetries in a system to conserved quantities. It states that for every continuous symmetry in a physical system, there exists a corresponding conserved quantity.

2. What is the significance of Noether's theorem?

Noether's theorem is significant because it provides a powerful tool for understanding the underlying structure and behavior of physical systems. It has been used to derive conservation laws in a wide range of fields, including classical mechanics, electromagnetism, and quantum field theory.

3. What is meant by the "conserved current arbitrariness" in Noether's theorem?

The "conserved current arbitrariness" refers to the fact that for a given symmetry in a system, there can be multiple conserved quantities that correspond to it. This is because there are many ways to express the same symmetry mathematically, and each expression leads to a different conserved quantity.

4. How is Noether's theorem used in practical applications?

Noether's theorem has been used in various practical applications in physics, such as in the derivation of the conservation of energy and momentum in classical mechanics, the conservation of charge in electromagnetism, and the conservation of baryon and lepton numbers in particle physics.

5. Are there any limitations to Noether's theorem?

While Noether's theorem is a powerful tool, it does have some limitations. It only applies to continuous symmetries, and there are some physical systems that do not exhibit continuous symmetries. Additionally, it does not take into account quantum effects, which may play a role in certain systems.

Similar threads

Replies
5
Views
400
Replies
3
Views
1K
Replies
1
Views
450
  • Quantum Physics
Replies
6
Views
4K
  • Quantum Physics
Replies
1
Views
813
Replies
1
Views
640
Replies
24
Views
2K
Replies
3
Views
796
  • Quantum Physics
Replies
1
Views
614
  • Classical Physics
Replies
4
Views
279
Back
Top